Đến nội dung

Pirates nội dung

Có 665 mục bởi Pirates (Tìm giới hạn từ 10-06-2020)



Sắp theo                Sắp xếp  

#227669 Dãy số

Đã gửi bởi Pirates on 29-01-2010 - 16:58 trong Phương trình - Hệ phương trình - Bất phương trình

1. Dãy số thực {$x_n$} được xác định bởi: $x_1 = \dfrac{29}{10}, x_{n + 1} = \dfrac{x_n}{\sqrt{x_n^{2} - 1}} + \sqrt{3}, n = 1, 2, 3...$
Hãy tìm số thực nhỏ hơn $x_{2k - 1}$ và lớn hơn $x_{2k}$ với mọi $k = 1, 2...$

2. Cmr nếu $a_1 > 2$ và $a_n = a^{2}_{n - 1} - 2$ thì:
$\dfrac{1}{a_1} + \dfrac{1}{a_1 a_2} + \dfrac{1}{a_1 a_2 a_3} + ... = \dfrac{1}{2}(a_1 - \sqrt{a_1^{2} - 4} )$



#224228 Pt và hpt lượng giác

Đã gửi bởi Pirates on 30-12-2009 - 17:11 trong Các bài toán Lượng giác khác

1. Giải phương trình:
$(2 + \sqrt{2})^{sin^{2}x} - (2 + \sqrt{2})^{cos^{2}x} + (2 - \sqrt{2})^{cos2x} = (1 + \dfrac{\sqrt{2}}{2})^{cos2x}$

2. Giải hệ phương trình:
$\left\{\begin{matrix} tg^{2}x + tg^{2}y + tg^{2}z = m^{2} \\ tg^{3}x + tg^{3}y + tg^{3}z = m^{3} \end{matrix}\right$



#207801 BĐT về đa thức hay

Đã gửi bởi Pirates on 02-08-2009 - 21:28 trong Bất đẳng thức và cực trị

1. Cho $a,b,c$ là 3 số mà phương trình: $x^{3} + ax^{2} + bx + c = 0$ có 3 nghiệm phân biệt. Chứng minh: $|27c + 2a^{3} - 9ab| < 2\sqrt{(a^{2} - 3b)^{3}}$

2. Cho đa thức: $f(x) = x^{4} + 4x^{3} - 8x + 1 - k$ có 4 nghiệm phân biệt. Chứng minh: $-3 < k < 6$



#219229 Mấy bài hay

Đã gửi bởi Pirates on 31-10-2009 - 17:19 trong Hình học

1. Chứng minh rằng diện tích tam giác có đỉnh là tâm các đường tròn bàng tiếp của một tam giác $ABC = \dfrac{2SR}{r}$, trong đó $S, R, r$ theo thứ tự là diện tích, bán kính đường tròn ngoại tiếp, bán kính đường tròn nội tiếp của tam giác $ABC$

2. Chứng minh rằng nếu $I$ là điểm Lemoine thì: $a^{2}AI^{2} + b^{2}BI^{2} + c^{2}CI^{2} = \dfrac{3a^{2}b^{2}c^{2}}{a^{2} + b^{2} + c^{2}}$



#226023 Phương trình.

Đã gửi bởi Pirates on 13-01-2010 - 16:53 trong Phương trình, hệ phương trình và bất phương trình

Đặt {$x$}$ = x - [x]$, ta có:

$0 \leq ${$x$}$ < 1$ và $[x] = x - ${$x$}.

$\Rightarrow x^{3} - (x - ${$x$}$) = 3$

hay $x^{3} - x = 3 -$($x$}

$\Rightarrow 2 < x^{3} - x \leq 3$

Với $x \geq 2$, ta có: $x^{3} - x = x(x^{2} - 1) \geq 2(4 - 1) = 6 > 3$

Với $x < -1$, ta có: $x^{2} - 1 > 0$ và $x^{3} - x = (x^{2} -1)x < 0 < 2$

Với $x = -1$, ta có: $x^{3} - x = 0 < 2$

Với $-1 < x \leq 0$, ta có: $x^{3} - x \leq -x < 1$

Với $0 < x \leq 1$, ta có: $x^{3} - x < x^{3} \leq 1$

Vậy $1 < x < 2$, nghĩa là $[x] = 1$. Ta có: $x^{3} - 1 = 3$ hay $x = \sqrt[3]{4}$



#205921 Bất đẳng thức hình học

Đã gửi bởi Pirates on 21-07-2009 - 13:35 trong Hình học

1. Chứng minh: Nếu trên các cạnh của tam giác ABC về phía ngoài dựng các tam giác đều với các tâm là D,E,F thì $S_{DEF} \geq S_{ABC}$
2. Gọi a,b,c là các cạnh của tam giác, q = a+b+c, S = ab+bc+ca. Chứng minh rằng: $3S < q^{2} < 4S$



#225644 [x] ?

Đã gửi bởi Pirates on 09-01-2010 - 19:43 trong Số học

Tính tổng $S = [\dfrac{1}{3}] + [\dfrac{2}{3}] + [\dfrac{2^{2}}{3}] + ... + [\dfrac{2^{2005}}{3}] $
Trong đó [x] là số nguyên lớn nhất không vượt quá x (phần nguyên của x)

Đây là một bài trong đề thi dề thi Olympic 30/4 toán 11 năm 2005, các bạn làm thử đi. :D

Bạn có thể post nguyên đề thi Olympic 30/4 năm 2005 lên được không?



#227487 BĐT lượng

Đã gửi bởi Pirates on 27-01-2010 - 17:28 trong Bất đẳng thức - Cực trị

Với mọi tam giác nhọn, cmr:

$(\dfrac{sin \alpha sin \beta}{sin \gamma})^{2} + (\dfrac{sin \beta sin \gamma}{sin \alpha})^{2} + (\dfrac{sin \gamma sin \alpha}{sin \beta})^{2} \geq \dfrac{9}{4}$



#223536 Vài bài trên tạp chí Kvant

Đã gửi bởi Pirates on 24-12-2009 - 17:10 trong Số học

1. Tìm các số tự nhiên $n$ để tồn tại các số hữu tỷ nhưng không nguyên $a, b$ sao cho $a + b$ và $a^{n} + b^{n}$ là số nguyên?

2. Với số tự nhiên $n$ nào thì tìm được bộ các số tự nhiên phân biệt sao cho:
$\dfrac{a_1}{a_2} + \dfrac{a_2}{a_3} + ... + \dfrac{a_n}{a_1}$ là số tự nhiên.

3. Tồn tại hay không bộ ba các số tự nhiên lớn hơn $10^{10}$ đôi một nguyên tố cùng nhau $(x ; y ; z)$ sao cho $x^{8} + y^{8} + z^{8} \vdots x^{4} + y^{4} + z^{4}$



#205919 Bài hay

Đã gửi bởi Pirates on 21-07-2009 - 13:20 trong Phương trình, hệ phương trình và bất phương trình

Giải pt: $\sqrt[3]{x+6} + \sqrt{x-1} = x^{2} - 1$



#218273 Chứng minh

Đã gửi bởi Pirates on 23-10-2009 - 17:57 trong Số học

Trục số thực càng đi xa về phía bên phải số 0 càng thấy các số nguyên tố xuất hiện thưa dần , có người nói đến một điểm nào đó sẽ chấm dứt tập số nguyên tố , nhận xét ấy đúng hay sai ? Bạn thử trả lời xem và hãy chứng minh để bảo vệ câu trả lời của mình .

Nói tóm lại, chúng ta sẽ đi chứng minh tồn tại vô hạn số nguyên tố.

Với $P_{n} = n! + 1$, $n \geq 1$
Ta có: $P_{n}$ có ít nhất một ước nguyên tố $p_{n}$. Nếu $p_{n} \leq n$ thì $p_{n} | n!$ và từ đó $p_{n} | (P_{n} - n!) = 1$ (mâu thuẫn). Vậy $p_{n} > n$.
Vậy với mọi số nguyên dương $n$, đều sẽ có số nguyên tố lớn hơn $n$, nên tồn tại vô hạn số nguyên tố.



#226030 Vô địch toán quốc tế 1972

Đã gửi bởi Pirates on 13-01-2010 - 17:55 trong Số học

Bài này cũ và khá hay, có một lời giải dùng phần nguyên. Ta sẽ chứng minh với mọi số $k$ nguyên tố, số các thừa số $k$ có trong $(2m)!(2n)! \geq$ số các thừa số $k$ có trong $m!n!(m + n)!$.



#223287 Tiếp tục làm nóng cho box Olympiad

Đã gửi bởi Pirates on 20-12-2009 - 16:24 trong Phương trình - Hệ phương trình - Bất phương trình

Thêm một vài bài:

4. Tìm tất cả các cặp hàm số $f: R -> R$ thỏa mãn điều kiện:
$f(x^{3} + 4y) + f(x + y) = g(x + y) \forall x, y \in R$

5. Cho các số $x, y, z$ dương. Chứng minh rằng với mọi $\Delta ABC$ ta đều có:
$x sin\dfrac{A}{2} + y sin\dfrac{B}{2} + z sin\dfrac{C}{2} \leq \dfrac{1}{2}(\dfrac{yz}{x} + \dfrac{xy}{z} + \dfrac{zx}{y})$

6. Cho dãy số $a_n$ xác định bởi $a_{n + 1} = \dfrac{a_n^{2} + c}{a_{n - 1}}$. Chứng minh rằng nếu $a_0, a_1$ và $\dfrac{a_0^{2} + a_1^{2} + c}{a_0a_1}$ là số nguyên thì $a_n$ nguyên với mọi $n$.



#223173 Tiếp tục làm nóng cho box Olympiad

Đã gửi bởi Pirates on 19-12-2009 - 08:30 trong Phương trình - Hệ phương trình - Bất phương trình

1. Chứng minh rằng: mỗi số nguyên lẻ đồng dư với đúng một số nguyên có dạng $(-1)^{\alpha}5^{\beta}$, trong đó $\alpha = 0$ hoặc $1, \beta$ là số nguyên thỏa mãn $0 \leq \beta \leq 2^{k - 2} - 1$ với $k \geq 3$.

2. Cho đa thức:
$P(x) = \sum\limits_{k = 0}^{n}C_n^{k}a_k^{k}x^{k} , a_k \geq 0 (k = 0, 1, ..., n)$
có các nghiệm đều thực. Chứng minh: $a_1 \geq a_2 \geq ... \geq a_n$.

3. Gọi $a_1, a_2, ..., a_n$ là các cạnh của một đa giác $n$ cạnh. Giả sử $(n - 1)p$ là chu vi của đa giác ấy. Giả thiết thêm rằng $a_i \leq p (i = 1, 2, ..., n)$.
Chứng minh: $\dfrac{n}{n - 1} \leq \sum\limits_{i=1}^{n}\dfrac{a_i}{\sum\limits_{k=1 , k \neq i}^{n}a_k} \leq \dfrac{n - 1}{n - 2}$



#204694 Làm bài này đi

Đã gửi bởi Pirates on 11-07-2009 - 08:05 trong Số học

1. Chứng minh rằng nếu ước nguyên tố nhỏ nhất của n là p, thì $x^{2} - n$ không phải là số chính phương nếu $ x > \dfrac{ n + p^{2} }{2p}$

2. Giả sử n là số nguyên dương. Chứng minh rằng lũy thừa của số nguyên tố p xuất hiện trong phân tích ra thừa số nguyên tố của n! là:
$ [n / p] + [n / p^{2}] + [n / p^{3}] + ... $



#211832 Một bài

Đã gửi bởi Pirates on 25-08-2009 - 13:31 trong Hình học

Cho tam giác $ABC$ nhọn, trên các tia đối của $BC, CA, AB$ lần lượt lấy các điểm $A_{1}, B_{1}, C_{1}$ sao cho tam giác $A_{1}B_{1}C_{1}$ đồng dạng với tam giác $ABC$. Chứng minh trực tâm của tam giác $A_{1}B_{1}C_{1}$ cũng là tâm đường tròn ngoại tiếp.



#224693 Lại vài bài

Đã gửi bởi Pirates on 02-01-2010 - 10:26 trong Bất đẳng thức - Cực trị

1. Cho $a_1, a_2, a_3, b_1, b_2, b_3, b > 0$. Cm:

$\dfrac{a_1 + b_1 + a_2 + b_2}{a_1 + b_1 + a_2 + b_2 + b_3 + b} + \dfrac{a_2 + b_2 + a_3 + b_3}{a_2 + b_2 + a_3 + b_3 + b_1 + b} > \dfrac{a_3 + b_3 + a_1 + b_1}{a_3 + b_3 + a_1 + b_1 + b_2 + b}$

2. Cho $a, b, c > 0$ thỏa $a + b + c = 4$. Cm:

$a^{\dfrac{3}{4}} + b^{\dfrac{3}{4}} + c^{\dfrac{3}{4}} > 2\sqrt{2}$



#211977 Một bài

Đã gửi bởi Pirates on 26-08-2009 - 13:34 trong Hình học

chắc là của ABC rồi


Uh, đề mình có cũng chỉ ghi như thế nên mình cho nguyên văn vậy, làm thử tâm đường tròn ngoại tiếp tam giác $ABC$ đi.



#206589 Làm bài này chơi

Đã gửi bởi Pirates on 26-07-2009 - 20:03 trong Bất đẳng thức và cực trị

1. Cho $a,b,c > 0$ và $n$ là một số tự nhiên dương.
Cm: $\dfrac{a^{n}}{b+c} + \dfrac{b^{n}}{c+a} + \dfrac{c^{n}}{a+b} \geq \dfrac{a^{n-1}+b^{n-1}+c^{n-1}}{2}$

2. Cho số thực không âm có tích bằng 1 $a_{1}, a_{2}, a_{3},... a_{k}$
Cm: $a_{1}^{m} + a_{2}^{m} + ... + a_{k}^{m} \geq a_{1}^{n} + a_{2}^{n} + ... + a_{k}^{n}$ với mọi $m \geq n$



#206601 Làm bài này chơi

Đã gửi bởi Pirates on 26-07-2009 - 20:47 trong Bất đẳng thức và cực trị

toàn dùng chebyshev cả :)


Để mấy em khác làm, dùng Côsi cho nó gọn...:D



#204697 Làm thử bài này đi

Đã gửi bởi Pirates on 11-07-2009 - 08:33 trong Hình học

Cho hình vuông ABCD cạnh AB = 1 nội tiếp trong đường tròn (O). Trên cung AB ta lấy điểm M sao cho $\widehat{AOM} = 30^{o}$. Gọi P là điểm đối xứng của M qua AB, Q đối xứng với P qua CD và H đối xứng với Q qua tâm O. Tính độ dài MH.



#223593 Bài số học hay đây

Đã gửi bởi Pirates on 25-12-2009 - 13:36 trong Số học

Chứng minh thế này:
S={1,2,...,p-1}. Mỗi s thuộc S có s' thuộc S sao cho s.s'=1(mod p) chú ý là s^2=1(mod p) khi s=1 hoặc p-1. Nên ghép cặp lại ta được. (p-1)!+1=(p-1)+1=0(mod p).
Cái này nhiều người chứng minh lắm r?#8220;i.

Hoàn chỉnh:

+ $p = 2$, ta có: $1! + 1 \equiv 0 (mod 2)$
+ $p > 2$:
Với $a = 1, 2, ..., p - 1$ ta có: $a^{p - 1} \equiv 1 (mod p) \Rightarrow a.a^{p - 2} \equiv 1 (mod p)$
Giả sử $a^{p - 2} \equiv b (mod p)$ thì $ab \equiv 1 (mod p)$.
Ta cm mỗi $a \in {1, 2, ..., p - 1}$ có duy nhất $b \in {1, 2, ..., p - 1}$ sao cho $ab \equiv 1 (mod p)$
Thật vậy giả sử có $c \in { 1, 2, ..., p - 1}$ sao cho $ac \equiv 1 (mod p)$
$\Rightarrow a( b - c) \vdots p \Rightarrow b = c$
Ta có: $(p - 1)! = 1.2...(p - 1) \equiv (p - 1) (mod p)$
Vậy $(p - 1)! \equiv -1 (mod p)$ hay $(p - 1)! + 1 \equiv 0 (mod p)$.



#226203 Bài nữa này

Đã gửi bởi Pirates on 15-01-2010 - 17:17 trong Số học

Tìm số nguyên tố p sao cho tổng tất cả các ước tự nhiên của $p^4$ là một số chính phương.
:D

Bài này thì đơn giản rồi

Ta có: $1 + p + p^{2} + p^{3} + p^{4} = k^{2}$

$\Rightarrow 4 + 4p + 4p^{2} + 4p^{3} + 4p^{4} = 4k^{2} = (2k)^{2}$

Ta lại có: $(2p^{2} + p)^{2} < (2k)^{2} < (2p^{2} + p + 2)^{2}$

$\Rightarrow 2k = 2p^{2} + p + 1$

$\Rightarrow p = 3$



#226097 Bài này hay đây

Đã gửi bởi Pirates on 14-01-2010 - 16:26 trong Số học

Chứng minh rằng số $2^{p}-1$ chỉ có ước nguyên tố dạng $2pk+1$, trong đó p là số nguyên tố.

Bài này không khó, chỉ cần áp dụng định lý Fermat và bổ đề $(k^{m} - 1 , k^{n} - 1) = k^{(m , n)} - 1$ là xong.



#205088 Một bài nữa

Đã gửi bởi Pirates on 15-07-2009 - 08:13 trong Phương pháp tọa độ trong mặt phẳng

Giả sử M là trung điểm của trung tuyến AD của $ \delta ABC $. Đường thẳng BM cắt AC tại N. Chứng minh rằng AB là tiếp tuyến của đường tròn ngoại tiếp $ \delta BCN $ khi và chỉ khi $ \dfrac{BM}{MN} = (\dfrac{AC}{AB})^{2} $.